korrigierte Lösungen
This commit is contained in:
parent
7c9bc94a1f
commit
10968a46e6
BIN
Logik und Logikprogrammierung - Übung.pdf
(Stored with Git LFS)
BIN
Logik und Logikprogrammierung - Übung.pdf
(Stored with Git LFS)
Binary file not shown.
@ -1,5 +1,5 @@
|
||||
\documentclass[10pt, a4paper]{exam}
|
||||
%\printanswers % Comment this line to hide the answers
|
||||
\printanswers % Comment this line to hide the answers
|
||||
\usepackage[utf8]{inputenc}
|
||||
\usepackage[T1]{fontenc}
|
||||
\usepackage[ngerman]{babel}
|
||||
@ -10,6 +10,10 @@
|
||||
\usepackage{bussproofs}
|
||||
\usepackage[many]{tcolorbox}
|
||||
|
||||
\usepackage{pifont}
|
||||
\newcommand{\cmark}{\ding{51}}
|
||||
\newcommand{\xmark}{\ding{55}}
|
||||
|
||||
% Don't print section numbers
|
||||
\setcounter{secnumdepth}{0}
|
||||
\qformat{\textbf{Aufgabe \thequestion}\dotfill \thepoints}
|
||||
@ -351,12 +355,20 @@
|
||||
|
||||
\begin{solution}
|
||||
|
||||
\begin{tabular}{c|c|c}
|
||||
$p_1$ & $p_1\rightarrow p_1$ & $p_1\rightarrow p_1 \Vdash p_1$ \\\hline
|
||||
w & w & w \\
|
||||
$0.5$ & w & w \\
|
||||
f & w & w \\
|
||||
\begin{tabular}{c|c|c|c|c|c}
|
||||
$p_1$ & $p_2$ & $p_1\rightarrow p_2$ & $\Gamma= inf\{p_1\rightarrow p_2, \varphi\}$ & $\Gamma\vdash p_2$ \\\hline
|
||||
0 & 0 & 1 & 0 & \cmark \\
|
||||
0 & $\frac{1}{2}$ & 1 & 0 & \xmark \\
|
||||
0 & 1 & 1 & 0 & \cmark \\
|
||||
$\frac{1}{2}$ & 0 & 0 & $\frac{1}{2}$ & \xmark \\
|
||||
$\frac{1}{2}$ & $\frac{1}{2}$ & $\frac{1}{2}$ & $\frac{1}{2}$ & \cmark \\
|
||||
$\frac{1}{2}$ & 1 & $\frac{1}{2}$ & 0 & \cmark \\
|
||||
1 & 0 & 0 & 0 & \cmark \\
|
||||
1 & $\frac{1}{2}$ & $\frac{1}{2}$ & $\frac{1}{2}$ & \cmark \\
|
||||
1 & 1 & 1 & 1 & \cmark
|
||||
\end{tabular}
|
||||
|
||||
B Tautologie, keine $K_3$ Tautologie
|
||||
\end{solution}
|
||||
|
||||
\subpart $\Gamma=\{p_1\rightarrow p_2, p_1\}, \varphi=p_2, W\in\{B,K_3\}$
|
||||
@ -398,11 +410,12 @@
|
||||
\subpart $\lnot(p_1\wedge \lnot p_1)$
|
||||
\begin{solution}
|
||||
|
||||
\begin{tabular}{c | c | c | c}
|
||||
$p_1$ & $\lnot p_1$ & $p_1\wedge \lnot p_1$ & $\lnot(p_1\wedge \lnot p_1)$ \\\hline
|
||||
w & f & f & w \\
|
||||
f & w & f & w
|
||||
\end{tabular}
|
||||
$B[\lnot(p_1\wedge \lnot p_2)]=1_b -inf\{B(p_1), -B(p_1)\}=1_B-0_B=1_B \Rightarrow$ B Tautologie
|
||||
|
||||
für $K_3$ betrachte Belegung $B=\frac{1}{2}$
|
||||
|
||||
$K[\lnot(p_1\wedge\lnot p_1)]= 1_b -inf\{B(p_1), -B(p_1)\} = 1-\frac{1}{2} \Rightarrow$ keine $K_3$ Tautologie
|
||||
%$B((p_1\vee p_2)\rightarrow(p_2\wedge p_3) = max(B(p_2\wedge p_3), 1-B(p_1\vee p_2)) = max(min(B(p_2),B(p_3)), 1-max(B(p_1),B(p_2))) = max(min(0.7,1),1-max(0.5,0.7)) = 0.7$
|
||||
|
||||
$\Rightarrow Tautologie$
|
||||
\end{solution}
|
||||
@ -533,21 +546,28 @@
|
||||
\begin{parts}
|
||||
\part Die Formelmenge $\{\varphi\}$ ist erfüllbar genau dann, wenn $\lnot\varphi$ kein Theorem ist.
|
||||
\begin{solution}
|
||||
$\varnothing\not\vdash\lnot\varphi \Vdash \{\varphi\}$
|
||||
%$\varnothing\not\vdash\lnot\varphi \Vdash \{\varphi\}$
|
||||
$\lnot\varphi$ Theorem $\leftrightarrow$ $\lnot\varphi$ B Tautologie
|
||||
|
||||
$\leftrightarrow B(\lnot \varphi)=1-B(\varphi)=1 \quad\forall$ B Bedingung
|
||||
|
||||
$\leftrightarrow \{\varphi\}$ erfüllbar
|
||||
\end{solution}
|
||||
|
||||
\part Wenn $\varphi$ eine F-Tautologie ist, dann ist $\bot$ eine Teilformel von $\varphi$.
|
||||
\begin{solution}
|
||||
$\Gamma\Vdash_F \rightarrow \varphi\rightarrow\bot$
|
||||
%$\Gamma\Vdash_F \rightarrow \varphi\rightarrow\bot$
|
||||
zeige $\bot$ keine Teilformel von $\varphi\rightarrow\varphi$ keine $K_3$ Tautologie. Idee: betrachte $K_3$ Belegung $B=\frac{1}{2}$, zeige $B(\varphi)=\frac{1}{2}$ induktiv
|
||||
\end{solution}
|
||||
|
||||
\part Das natürliche Schließen ist auch ohne die Regel $(\bot)$ vollständig.
|
||||
\begin{solution}
|
||||
|
||||
ersetzte Deduktion durch Falsum
|
||||
\end{solution}
|
||||
|
||||
\part Für jede aussagenlogische Formel $\varphi$ gibt es unendlich viele, paarweise verschiedene, äquivalente Formeln.
|
||||
\begin{solution}
|
||||
definiere Folge $(\varphi)_{i=N}$ mit $\varphi_1=\varphi$ und $\varphi_{i+1}=\varphi_i +\varphi$
|
||||
|
||||
Idee: Kombination der Menge mit beliebige u.u. unendlich vielen Formeln $\lnot\bot$
|
||||
|
||||
@ -579,25 +599,31 @@
|
||||
\part Überprüfen Sie mittels Makierungsalgorithmus, ob die unten angegebene Folgerung gilt.
|
||||
$$p_1\wedge (p_2\vee \lnot p_3\vee \lnot p_5)\wedge (\lnot p_1\vee p_3)\wedge (\lnot p_3\vee p_4)\wedge (\lnot p_1\vee p_2)\Vdash p_5$$
|
||||
\begin{solution}
|
||||
Markierungsalgorithmus M:
|
||||
\begin{itemize}
|
||||
\item Eingabe: Menge von Hornklauseln (Negationen auf linke Seite, Positive auf Rechte)
|
||||
\item Ausgabe: $M(\Gamma)=$ erfüllbar $\leftrightarrow \Gamma$ erfüllbar
|
||||
\item Grundlage: $\Gamma\Vdash\varphi \leftrightarrow \Gamma\cup\{\lnot\varphi\}$ unerfüllbar
|
||||
\end{itemize}
|
||||
\begin{itemize}
|
||||
\item $\{ (p_1), (p_2\vee \lnot p_3\vee \lnot p_5), (\lnot p_1\vee p_3), (\lnot p_3\vee p_4), (\lnot p_1\vee p_2), \lnot p_5\}$
|
||||
\item Umformen
|
||||
\item Hornklauseln
|
||||
\begin{enumerate}
|
||||
\item $\lnot\bot\rightarrow p_1$
|
||||
\item $p_3\wedge p_5\rightarrow p_2$
|
||||
\item $p_1\rightarrow p_3$
|
||||
\item $p_3\rightarrow p_4$
|
||||
\item $p_1\rightarrow p_2$
|
||||
\item $p_5\rightarrow \bot$
|
||||
\end{enumerate}
|
||||
\item Markier-Algorithmus
|
||||
\begin{itemize}
|
||||
\item $p_1\equiv \varnothing\rightarrow p_1$
|
||||
\item $\lnot p_5\equiv p_5\rightarrow \bot$
|
||||
\item $\lnot p_1\vee p_3 \equiv p_1\rightarrow p_3 = \{p_1\}\rightarrow p_3$
|
||||
\item $\lnot p_3\vee p_4 \equiv p_3\rightarrow p_4 = \{p_3\}\rightarrow p_4$
|
||||
\item $\lnot p_1\vee p_2 \equiv p_1\rightarrow p_2 = \{p_1\}\rightarrow p_2$
|
||||
\item $p_2\vee\lnot p_3\vee\lnot p_5 \equiv p_3\wedge p_5\rightarrow p_2 = \{p_3,p_5\}\rightarrow p_2$
|
||||
\item $p_1$ für 1.
|
||||
\item $p_3,p_4$ für 3.,5.
|
||||
\item $p_4$ für 4.
|
||||
\item keine Terme übrig, allg. Terminiert mit ,,erfüllbar''
|
||||
\end{itemize}
|
||||
\item $\{\varnothing\rightarrow p_1, \{p_5\}\rightarrow\bot, \{p_1\}\rightarrow p_3, \{p_3\}\rightarrow p_4, \{p_1\}\rightarrow p_2, \{p_3,p_5\}\rightarrow p_2$
|
||||
\item Algorithmus
|
||||
\begin{itemize}
|
||||
\item Markiere $p_1$ da $\varnothing\rightarrow p_1$
|
||||
\item Markiere $p_3,p_2$ da $\{p_1\}\rightarrow p_3, \{p_1\}\rightarrow p_2$
|
||||
\item Markiere $p_4$ da $\{p_3\}\rightarrow p_4$
|
||||
\end{itemize}
|
||||
\item Algorithmus bricht ab, Formel ist erfüllbar
|
||||
\item erfüllbar $\rightarrow$ Folgerung falsch $\rightarrow \Gamma\not\Vdash\varphi$
|
||||
\end{itemize}
|
||||
\end{solution}
|
||||
|
||||
@ -605,25 +631,24 @@
|
||||
$$(p_1\wedge\lnot p_2\wedge p_3)\vee(p_4\wedge\lnot p_1)\vee(p_2\wedge\lnot p_4)\vee\lnot p_2\vee p_4$$
|
||||
\begin{solution}
|
||||
\begin{itemize}
|
||||
\item $\lnot\phi = \lnot((p_1\wedge\lnot p_2\wedge p_3)\vee(p_4\wedge\lnot p_1)\vee(p_2\wedge\lnot p_4)\vee\lnot p_2\vee p_4)$
|
||||
\item $= (\lnot p_1\vee p_2\lnot p_3)\wedge(\lnot p_4\vee p_1)\wedge(\lnot p_2\vee p_4)\wedge p_2 \wedge \lnot p_4$
|
||||
\item $\{(\lnot p_1\vee p_2\lnot p_3),(\lnot p_4\vee p_1),(\lnot p_2\vee p_4),p_2,\lnot p_4)\}$ keine Menge von Hornklauseln
|
||||
\item Umformen
|
||||
\item $\varphi$ Tautologie $\leftrightarrow \{\lnot\varphi\}$ unerfüllbar
|
||||
\item $\phi = (p_1\wedge\lnot p_2\wedge p_3)\vee(p_4\wedge\lnot p_1)\vee(p_2\wedge\lnot p_4)\vee\lnot p_2\vee p_4$
|
||||
\item $\lnot\phi=(\lnot p_1\vee p_2\vee\lnot p_3)\wedge(\lnot p_4\vee p_1)\wedge(\lnot p_2\vee p_4)\wedge p_2\wedge\lnot p_4$
|
||||
\item Hornklauseln
|
||||
\begin{enumerate}
|
||||
\item $p_1 \wedge p_3 \rightarrow p_2$
|
||||
\item $p_4 \rightarrow p_1$
|
||||
\item $p_2 \rightarrow p_4$
|
||||
\item $\lnot\varnothing\rightarrow p_2$
|
||||
\item $p_4\rightarrow\bot$
|
||||
\end{enumerate}
|
||||
\item Markier-Algorithmus
|
||||
\begin{itemize}
|
||||
\item $(\lnot p_1\vee p_2\lnot p_3) \equiv p_1\wedge p_3\rightarrow p_2 = \{p_1,p_3\}\rightarrow p_2$
|
||||
\item $(\lnot p_4\vee p_1)\equiv p_4\rightarrow p_1 = \{p_4\}\rightarrow p_1$
|
||||
\item $(\lnot p_2\vee p_4)\equiv p_2\rightarrow p_4 = \{p_2\}\rightarrow p_4$
|
||||
\item $p_2 \equiv \varnothing\rightarrow p_2$
|
||||
\item $\lnot p_4 \equiv p_4\rightarrow\bot = \{p_4\}\rightarrow\bot$
|
||||
\item $p_2$ für 4.
|
||||
\item $p_4$ für 3.
|
||||
\item wegen 5. bricht Algorithmus ab mit unerfüllbar
|
||||
\end{itemize}
|
||||
\item $\{\{p_1,p_3\}\rightarrow p_2, \{p_4\}\rightarrow p_1, \{p_2\}\rightarrow p_4, \varnothing\rightarrow p_2, \{p_4\}\rightarrow\bot\}$
|
||||
\item Algorithmus
|
||||
\begin{itemize}
|
||||
\item Markiere $p_2$ wegen $\varnothing\rightarrow p_2$
|
||||
\item Markiere $p_4$ wegen $p_2\rightarrow p_4$
|
||||
\item Markiere $p_1$, wegen $p_4\rightarrow p_1$
|
||||
\end{itemize}
|
||||
\item Ausgabe unerfüllbar da $p_4\rightarrow\bot$, also ist es Tautologie
|
||||
\item Ausgabe unerfüllbar da $p_4\rightarrow\bot$, also ist $\phi$ Tautologie
|
||||
\end{itemize}
|
||||
\end{solution}
|
||||
\end{parts}
|
||||
@ -633,31 +658,29 @@
|
||||
\part Überprüfen Sie mittels SLD-Resolution, ob die unten angegebene Folgerung gilt.
|
||||
$$p_1\wedge(\lnot p_1\vee\lnot p_2\vee p_4)\wedge(\lnot p_1\vee p_3\vee\lnot p_4)\wedge(p_6\vee\lnot p_3)\wedge(\lnot p_2\vee p_5\vee\lnot p_6)\Vdash\lnot p_2\vee(p_4\wedge p_5)$$
|
||||
\begin{solution}
|
||||
SLD Resolution für $B(M_1\rightarrow\bot, M_2\rightarrow\bot,...,M_n\rightarrow\bot)$ mit $M_n\rightarrow\bot\in\Gamma$ und $M_{ind}=M_n\wedge\{\varphi\}\wedge N$ für $N\rightarrow\varphi\in\Gamma$. $M_n=\varnothing\Leftrightarrow\Gamma$ unerfüllbar
|
||||
|
||||
\begin{itemize}
|
||||
\item $\{(p_1),(\lnot p_1\vee\lnot p_2\vee p_4),(\lnot p_1\vee p_3\vee\lnot p_4),(p_6\vee\lnot p_3),(\lnot p_2\vee p_5\vee\lnot p_6),(\lnot p_2\vee (p_4\wedge p_5)\}$
|
||||
\item Umformen
|
||||
\item Horn-Klauseln
|
||||
\begin{enumerate}
|
||||
\item $\varnothing \rightarrow p_1$
|
||||
\item $p_1 \wedge p_2\rightarrow p_4$
|
||||
\item $p_1 \wedge p_4\rightarrow p_3$
|
||||
\item $p_3 \rightarrow p_6$
|
||||
\item $p_6 \wedge p_2 \rightarrow p_5$
|
||||
\item $\lnot\bot\rightarrow p_2$
|
||||
\item $p_4\wedge p_5\rightarrow \bot$
|
||||
\end{enumerate}
|
||||
\item SLD Mengen
|
||||
\begin{itemize}
|
||||
\item $p_1 \equiv \varnothing \rightarrow p_1$
|
||||
\item $(\lnot p_1\vee \lnot p_2 \vee p_4) \equiv p_1\wedge p_2\rightarrow p_4 = \{p_1,p_2\}\rightarrow p_4$
|
||||
\item $(\lnot p_1\vee p_3\vee\lnot p_4)\equiv p_1\wedge p_4\rightarrow p_3 = \{p_1,p_4\}\rightarrow p_3$
|
||||
\item $(p_6\vee\lnot p_3)\equiv p_3\rightarrow p_6 = \{p_3\}\rightarrow p_6$
|
||||
\item $(\lnot p_2\vee p_5\vee\lnot p_6) \equiv p_6\wedge p_2\rightarrow p_5 = \{p_6,p_2\}\rightarrow p_5$
|
||||
\item $p_4\wedge p_5 \rightarrow \bot = \{p_4,p_5\}\rightarrow \bot$
|
||||
\item $\lnot(\lnot p_2\vee(p_4\wedge p_5))=p_2\wedge \lnot(p_4 \wedge p_5) \equiv \varnothing\rightarrow p_2$
|
||||
\item aus 7.: $M_1=\{p_4,p_5\}$
|
||||
\item aus 5.: $M_2=\{p_2,p_4,p_6\}$
|
||||
\item aus 4.: $M_3=\{p_2,p_3,p_4\}$
|
||||
\item aus 3.: $M_4=\{p_1,p_2,p_4\}$
|
||||
\item aus 2.: $M_5=\{p_1,p_2\}$
|
||||
\item aus 1.+6.: $M_6=\varnothing$
|
||||
\end{itemize}
|
||||
\item SLD
|
||||
\begin{itemize}
|
||||
\item $M_0=\{p_5,p_4\}$
|
||||
\item $M_1=M_0\backslash p_5\cup \{p_6,p_2\}=\{p_2,p_4,p_6\}$
|
||||
\item $M_2=M_1\backslash p_4\cup \{p_1,p_2\} = \{p_1,p_2,p_6\}$
|
||||
\item $M_3=M_2\backslash p_6\cup \{p_3\} = \{p_1,p_2,p_3\}$
|
||||
\item $M_4=M_3\backslash p_3\cup \{p_1,p_4\} = \{p_1,p_2,p_4\}$
|
||||
\item $M_5=M_4\backslash p_4\cup \{p_1,p_2\} = \{p_1, p_2\}$
|
||||
\item $M_6=M_5\backslash p_1\cup \{\varnothing\} = \{p_2\}$
|
||||
\item $M_7=M_6\backslash p_2\cup \{\varnothing\} = \varnothing$
|
||||
\end{itemize}
|
||||
\item = nicht erfüllbar
|
||||
\item $M=\varnothing\rightarrow \Gamma$ unerfüllbar $\rightarrow \varphi_{links}\Vdash\varphi_{rechts}$
|
||||
\end{itemize}
|
||||
\end{solution}
|
||||
|
||||
@ -665,25 +688,24 @@
|
||||
$$(p_2\wedge\lnot p_1\wedge p_3)\vee(p_4\wedge p_1\wedge p_3)\vee(\lnot p_4\wedge p_1\wedge p_2)\vee\lnot p_3\vee\lnot p_2$$
|
||||
\begin{solution}
|
||||
\begin{itemize}
|
||||
\item $\lnot((p_2\wedge\lnot p_1\wedge p_3)\vee(p_4\wedge p_1\wedge p_3)\vee(\lnot p_4\wedge p_1\wedge p_2)\vee\lnot p_3\vee\lnot p_2)$
|
||||
\item $(\lnot p_2\vee p_1 \vee \lnot p_3)\wedge(\lnot p_4\vee\lnot p_1 \vee\lnot p_3)\wedge(p_4\vee\lnot p_1\vee\lnot p_2)\wedge p_3 \wedge p_2$
|
||||
\item Umformen
|
||||
\item Invertiere Formel für SLD
|
||||
\item Horn-Klauseln
|
||||
\begin{enumerate}
|
||||
\item $p_2 \wedge p_3 \rightarrow p_1$
|
||||
\item $p_1 \wedge p_3 \wedge p_4 \rightarrow\bot$
|
||||
\item $p_1 \wedge p_2 \rightarrow p_4$
|
||||
\item $\lnot\bot\rightarrow p_3$
|
||||
\item $\lnot\bot\rightarrow p_2$
|
||||
\end{enumerate}
|
||||
\item SLD Mengen
|
||||
\begin{itemize}
|
||||
\item $(\lnot p_2\vee p_1 \vee \lnot p_3)\equiv p_2\wedge p_3 \rightarrow p_1 =\{p_2,p_3\}\rightarrow p_1$
|
||||
\item $(\lnot p_4\vee\lnot p_1 \vee\lnot p_3)\equiv p_4\wedge p_1\wedge p_3\rightarrow\bot = \{p_1,p_3,p_4\}\rightarrow\bot$
|
||||
\item $(p_4\vee\lnot p_1\vee\lnot p_2)\equiv p_1\wedge p_2\rightarrow p_4 = \{p_1,p_2\}\rightarrow p_4$
|
||||
\item $p_3\equiv \varnothing \rightarrow p_3$
|
||||
\item $p_2\equiv \varnothing \rightarrow p_2$
|
||||
\item aus 2.: $M_1=\{p_1,p_3,p_4\}$
|
||||
\item aus 1.: $M_2=\{p_2,p_3,P_4\}$
|
||||
\item aus 3.: $M_3=\{p_1,p_2,p_3\}$
|
||||
\item aus 1.: $M_4=\{p_2,p_3\}$
|
||||
\item aus 4.+5.: $M_5=\varnothing$
|
||||
\end{itemize}
|
||||
\item SLD
|
||||
\begin{itemize}
|
||||
\item $M_0 = \{p_1,p_3,p_4\}$
|
||||
\item $M_1 = M_0\backslash p_4\cup\{p_1,p_2\} = \{p_1,p_2,p_3\}$
|
||||
\item $M_2 = M_1\backslash p_1\cup\{p_2,p_3\} = \{p_2,p_3\}$
|
||||
\item $M_3 = M_2\backslash p_2\cup\{\varnothing\} = \{p_3\}$
|
||||
\item $M_4 = M_3\backslash p_3\cup\{\varnothing\} = \varnothing$
|
||||
\end{itemize}
|
||||
\item = nicht erfüllbar; ist Tautologie
|
||||
\item $M=\varnothing\rightarrow \{\lnot\varphi\}$ unerfüllbar $\rightarrow \varphi$ Tautologie
|
||||
\end{itemize}
|
||||
\end{solution}
|
||||
\end{parts}
|
||||
@ -691,17 +713,17 @@
|
||||
\begin{parts}
|
||||
\part $a\rightarrow b \equiv \lnot b\rightarrow \lnot a$
|
||||
\begin{solution}
|
||||
|
||||
$a\rightarrow b \equiv \lnot a\vee b\equiv b\vee\lnot a\equiv \lnot\lnot b\vee\lnot a\equiv \lnot b\rightarrow \lnot a$
|
||||
\end{solution}
|
||||
|
||||
\part $a\vee (a\wedge b)\equiv a$
|
||||
\begin{solution}
|
||||
|
||||
nutze $\alpha=\beta$ gdw $\alpha\leftrightarrow\beta$ ist Theorem, gdw $\alpha\rightarrow\beta, \beta\rightarrow\alpha$ Theoreme
|
||||
\end{solution}
|
||||
|
||||
\part $\lnot a \rightarrow \bot \equiv a$
|
||||
\begin{solution}
|
||||
|
||||
$\lnot a\rightarrow \bot \equiv \lnot\lnot a\vee \bot\equiv a\vee \bot\equiv a\vee(a\wedge\lnot a)\equiv a$
|
||||
\end{solution}
|
||||
\end{parts}
|
||||
|
||||
@ -715,17 +737,19 @@
|
||||
\begin{parts}
|
||||
\part Aus $\Gamma\not\Vdash_W \phi$ folgt $\Gamma\Vdash_w\lnot\phi$ für jeden Wahrheitswertebereich $W$.
|
||||
\begin{solution}
|
||||
$\Gamma=\{\lnot\bot\}$, $\varphi=p$, $W=K_3$, $K_3$ Belegung $B=\frac{1}{2}$
|
||||
|
||||
$\rightarrow inf\ B[\Gamma]=1$ und $B(\varphi)=B(p)=\frac{1}{2}\rightarrow f\not\Vdash_{k_3} \varphi, \lnot\varphi$
|
||||
\end{solution}
|
||||
|
||||
\part Es gibt eine Menge aussagenlogischer Formeln $\Gamma$ und eine Formel $\phi$ mit $\Gamma\vdash\phi$ und $\Gamma\vdash\lnot\phi$.
|
||||
\begin{solution}
|
||||
|
||||
$\Gamma=\{\bot\}, \varphi$ beliebig, z.b. $\varphi=\bot$
|
||||
\end{solution}
|
||||
|
||||
\part Angenommen, es gäbe eine aussagenlogische Formel $\phi$ mit $\varnothing\vdash\phi$ und $\varnothing\vdash\lnot\phi$. Dann ist jede aussagenlogische Formel ein Theorem.
|
||||
\begin{solution}
|
||||
|
||||
betrachte $\psi$, ist Deduktion für $\varnothing\rightarrow\psi\rightarrow\psi$ Theorem
|
||||
\end{solution}
|
||||
\end{parts}
|
||||
|
||||
@ -746,22 +770,22 @@
|
||||
\begin{parts}
|
||||
\part $\forall x:Q(x,x)\rightarrow\exists x:Q(x,y)$
|
||||
\begin{solution}
|
||||
kein Satz, y ist nicht gebunden
|
||||
y frei $\rightarrow$ kein Satz
|
||||
\end{solution}
|
||||
|
||||
\part $P(f(x))\rightarrow\exists x:P(x)$
|
||||
\begin{solution}
|
||||
kein Satz, denn das erste x ist nicht gebunden
|
||||
x vorn frei $\rightarrow$ kein Satz
|
||||
\end{solution}
|
||||
|
||||
\part $P(a)\vee P(f(a))$
|
||||
\begin{solution}
|
||||
Satz, da es keine (freien) Variablen in der Formel gibt
|
||||
keine freien Variablen $\rightarrow$ ist Satz
|
||||
\end{solution}
|
||||
|
||||
\part $\exists z:(Q(z,x)\vee Q(y,z))\rightarrow\exists y:(Q(x,y)\wedge Q(x,z))$
|
||||
\begin{solution}
|
||||
kein Satz, da z.B. x nicht gebunden ist
|
||||
y vorn frei, z hinten frei $\rightarrow$ kein Satz
|
||||
\end{solution}
|
||||
\end{parts}
|
||||
|
||||
@ -784,13 +808,13 @@
|
||||
\end{center}
|
||||
\begin{solution}
|
||||
|
||||
a) $E_a=\exists x\exists y(\lnot x=y\wedge \lnot E(x,y))$ also $G_1\Vdash E_a$ und $G_2\not\Vdash E_a$
|
||||
a) $\exists u,u': (v\not=v' \wedge \lnot E(v,v'))$
|
||||
|
||||
b) $E_b=\exists x\exists y\exists z(x\not= y\wedge x\not= z \wedge y\not= z)$
|
||||
b) $\exists v,w,w': (E(v,w) \wedge E(v,w') \wedge E(w,w') \wedge v\not=w\not=w')$
|
||||
|
||||
c) $E_c=\forall x(\lnot E(x,x))$
|
||||
c) $\lnot\exists v: E(v,v))$
|
||||
|
||||
d) $E_d=$
|
||||
d) $\exists v: \forall v': \lnot E(v,v')$
|
||||
\end{solution}
|
||||
|
||||
\question Sei $\Gamma$ die Signatur bestehend aus einem zwei-stelligen Relationssymbol $\in$. Für eine Menge von Mengen $M$ definieren wir die Struktur $S$ mit $U_S=M$ und $\in^S=\in$. Geben Sie für jede der folgenden Aussagen eine Formel an, die diese beschreibt.
|
||||
|
Loading…
Reference in New Issue
Block a user